You are on page 1of 36

Problem Set 0 (Recap)

Corporate Finance 2.5 – Spring 2020


Question 1:

Consider a project that is expected to generate cash flow 𝐶𝐹 each year. Assuming a
constant discount factor 𝑟 for all periods, derive the PV formula
a) if the project runs in perpetuity.
b) if the project only pays cash flows for a finite number of x periods.
Hint: we want to derive the expressions given on lecture slide “Discounting: Useful
Present-Value formulas”.

Question 2:

You work on the financial planning for the next year. Your working capital policies are to
maintain Accounts Receivable Days of 45, Inventory Days of 24, and Accounts Payable
Days of 30. Quarterly forecasts for sales and COGS are provided by the table below (in
th). Estimate your quarterly working capital needs.

Quarter
1 2 3 4
Sales $225 $300 $300 $284
Cost of goods sold $168.75 $225 $225 $213

Question 3:

A firm is considering acquiring a new machine that will save it $1000 in the first year of
its operation. Due to wear and tear the amount of cost saving the machine produces will
decline at a rate of 2% per year in perpetuity. What is the present value of the total
savings generated by the machine if the applicable discount rate is 5% per annum?

Question 4:

Genetec Inc. is considering an investment in a new operations management system


which will last for 5 years. If implemented, the new system will increase Genetec's sales
by $10,000 in the first year of the project. The sales are forecast to grow at 10% per year
in the subsequent years during the life of the system. This system will also lower the
firm's costs by $280,000 per year. It will also lead to a one-time reduction in NWC by
$125,000 today. However, at the end of the project, the NWC has to be brought back to
its original level. The cost of the system is $840,000 which will be depreciated using the
straight-line method down to book value of 0 over the useful life of the system. At the
end of its life, the system can be sold for $75,000. Should Genetec undertake this project
if the firm's tax rate is 35% and the cost of capital is 20%?
Solution – Problem Set 0
Corporate Finance 2.5 – Spring 2020

Question 1:

a) 𝑃𝑉 = +( )
+ ⋯ Multiply both sides by (1 + 𝑟) and rewrite the RHS.
⇔ (1 + 𝑟) ∙ 𝑃𝑉 = 𝐶 + +( )
+ ⋯ = 𝐶 + 𝑃𝑉 Then simply solve for PV.
⇒ 𝑃𝑉 =
Comment: Be aware of the precise timing for this formula. For this equation to
hold, the first cash flow needs to be discounted by one entire period, etc. What
would be the present value formula if the first cash flow would not be
discounted?

b) Take the PV of the perpetuity and subtract the value of a perpetuity that starts
after the last cash flow in period x.
𝑃𝑉 = − ∙ ( )

Question 2:

Quarter
1 2 3 4
AR 45*$225/90=112.5 $150 $150 $142
INV 24*$168.75/90=45 $60 $60 $56.8
AP 30*$168.75/90=56.25 $75 $75 $71
NWC $101.25 $135 $135 $127.8

Question 3:

Timeline:
0 1 2 3

1,000 1,000(1 – 0.02) 1,000(1 – 0.02)2


We must value a growing perpetuity with a negative growth rate of -0.02:
1, 000
PV   $14, 285.71
0.05  0.02

Question 4:

Year 0 Year 1 Year 2 Year 3 Year 4 Year 5


Change in revenues 10000 11000 12100 13310 14641
Change in costs 280000 280000 280000 280000 280000
Change in depreciation -168000 -168000 -168000 -168000 -168000
Change in EBIT 122000 123000 124100 125310 126641

Change in tax (35%) -42700 -43050 -43435 -43858.5 -44324.4

Add back change in


depreciation 168000 168000 168000 168000 168000

Change in NWC 125000 -125000


Change in Capex -840000 48750

Project FCF -715000 247300 247950 248665 249451.5 174066.7

PV -715000 206083.3 172187.5 143903.4 120298.8 69953.48

NPV -2573.57
Problem Set 1
Corporate Finance 2.5 – Spring 2020
Question 1:

Proof Modigliani-Miller’s Theorem by showing that there have to be arbitrage


opportunities in a perfect capital market if two firms with identical cash flows but
different capital structures have different valuations.
Assume that Firm 1 if financed by equity 𝐸 and risk-free debt 𝐷 . Firm to is an all-equity
firm with equity 𝐸 .

Question 2:

Consider two Firms i = A, B that generate an identical cash flow X = 110 at t = 1 but have
different capital structures. Firm A is all equity financed, whereas firm B has debt
maturing at t = 1 with a principal of K = 40. The discount rate is 0.1 which is also the rate
at which money can be borrowed or lent by households and firms. The value of firm i at
t=0 is denoted by 𝑉 . The value of firm i’s equity and debt at t = 0 are respectively 𝐸 and
𝐷 . Finally, assume that short sales are possible.1
a) What are the respective cash flows to the shareholders of firm A and B at t = 1?
Compute and compare 𝐷 , 𝐸 and 𝐸 ?
b) Suppose that the market value of firm A’s equity at t = 0 is 𝐸 as derived in (a) and
the market value of firm B’s equity is 𝐸 = 54. Consider an investor who has own
wealth W = 54 to invest. Suppose she decides to buy all of firm B’s equity. What is
her final payoff? What is her final payoff, if she buys all of firm A’s equity?
c) Propose an arbitrage strategy for the investor in (b). If we assume that rational
investors do not leave arbitrage opportunities unexploited, what should be the
equilibrium values for 𝐸 and 𝐸 as well as 𝑉 and 𝑉 ? Which general theorem is
this an example of? State the essence of the theorem in one sentence and list the
assumptions under which it is derived.

Question 3:

Global Pistons (GP) has common stock with a market value of $200 million and debt
with a value of $100 million. Investors expect a 15% return on the stock and a 6% return
on the debt. Assume perfect capital markets.
a) Suppose GP issues $100 million of new stock to buy back the debt. What is the
expected return of the stock after this transaction?
b) Suppose instead GP issues $50 million of new debt to repurchase stock.
i) If the risk of the debt does not change, what is the expected return of the
stock after this transaction
ii) If the risk of the debt increases, would the expected return of the stock be
higher or lower than in part (i)?

1
Short sale: Borrowing a security and selling it at t=0, with the understanding that
the security or the associated cash flow must be returned to the original owner at
t=1.
Question 4:

Pelamed Pharmaceuticals has EBIT of $325 million in 2006. In addition, Pelamed has
interest expenses of $125 million and a corporate tax rate of 40%.
a) What is Pelamed’s 2006 net income?
b) What is the total of Pelamed’s 2006 net income and interest payments?
c) If Pelamed had no interest expenses, what would its 2006 net income be? How
does it compare to your answer in part (b)?
d) What is the amount of Pelamed’s interest tax shield in 2006?

Question 5:

Colt Systems will have EBIT this coming year of $15 million. It will also spend $6 million
on total capital expenditures and increases in net working capital, and have $3 million in
depreciation expenses. Colt is currently an all-equity firm with a corporate tax rate of
35% and a cost of capital of 10%.
a) If Colt is expected to grow by 8.5% per year, what is the market value of its equity
today?
b) If the interest rate on its debt is 8%, how much can Colt borrow now and still
have nonnegative net income this coming year?
c) Is there a tax incentive for Colt to choose a debt-to-value ratio that exceeds 50%?
Explain.

Question 6:

Gladstone Corporation is about to launch a new product. Depending on the success of


the new product, Gladstone may have one of four values next year: $150 million, $135
million, $95 million, or $80 million. These outcomes are all equally likely, and this risk is
diversifiable. Gladstone will not make any payouts to investors during the year. Suppose
the risk-free interest rate is 5% and assume perfect capital markets.
a) What is the initial value of Gladstone’s equity without leverage?

Now suppose Gladstone has zero-coupon debt with a $100 million face value due next
year.

b) What is the initial value of Gladstone’s debt?


c) What is the yield-to-maturity of the debt? What is its expected return?
d) What is the initial value of Gladstone’s equity? What is Gladstone’s total value
with leverage?

Question 7:

Marpor Industries has no debt and expects to generate free cash flows of $16 million
each year. Marpor believes that if it permanently increases its level of debt to $40
million, the risk of financial distress may cause it to lose some customers and receive
less favorable terms from its suppliers. As a result, Marpor’s expected free cash flows
with debt will be only $15 million per year. Suppose Marpor’s tax rate is 35%, the risk-
free rate is 5%, the expected return of the market is 15%, and the beta of Marpor’s free
cash flows is 1.10 (with or without leverage).
a) Estimate Marpor’s value without leverage.
b) Estimate Marpor’s value with the new leverage.

Question 8:

Acort Industries owns assets that will have an 80% probability of having a market value
of $50million in one year. There is a 20% chance that the assets will be worth only $20
million. The current risk-free rate is 5%, and Acort’s assets have a cost of capital of 10%.
a) If Acort is unlevered, what is the current market value of its equity?
b) Suppose instead that Acort has debt with a face value of $20 million due in one
year. According to MM, what is the value of Acort’s equity in this case?
c) What is the expected return of Acort’s equity without leverage? What is the
expected return of Acort’s equity with leverage?
d) What is the lowest possible realized return of Acort’s equity with and without
leverage?
Solution – Problem Set 1
Corporate Finance 2.5 – Spring 2020
Question 1:

Denote the risky cash flow of firms by 𝑋 and the risk-free rate by 𝑟 . 𝑉 = 𝐸 + 𝐷 and
𝑉 =𝐸 .

Suppose that 𝑉 < 𝑉 :

Strategy: “Sell the expensive asset, and buy the cheap…”


Short sell 𝛼𝑉 and buy 𝛼 𝐷 + 𝛼 𝐸 = 𝛼𝑉 , which is a self-financing strategy.

The investor realizes the following cash flow at the end of the period:

𝛼𝑉 𝛼𝑉 𝑉
𝐷 1+𝑟 + 𝑋−𝐷 1+𝑟 − 𝛼𝑋 = 𝛼 −1 𝑋 >0
𝑉 𝑉 𝑉

Suppose that 𝑉 > 𝑉 :

Short sell 𝛼𝐸 , borrow 𝛼𝐷 , and buy 𝐸 , which is again a self-financing strategy.

The investor realizes the following cash flow at the end of the period:

𝛼𝑉 𝑉
𝑋 − 𝛼𝐷 1 + 𝑟 − 𝛼 𝑋 − 𝐷 1 + 𝑟 =𝛼 −1 𝑋 >0
𝑉 𝑉

Question 2:

a) The t = 1 cash flow to the shareholders of firm A is the entire cash flow 110. Firm
B shareholders only receive the firm’s cash flow net of the debt repayment and
interest, i.e. 110 − 40(1 + 𝑟) = 110 − 44 = 66. The value of firm A’s equity at t
= 0 is 𝐸 = 110/1.1 = 100. The value of firm B’s debt at t = 0 is 𝐷 = 44/1.1 =
40. The value of firm B’s equity is 𝐸 = . = 60. The firms’ values are identical:
𝑉 = 𝑉 = 100.

b) It is assumed that 𝐸 = 100 and 𝐸 = 54. We consider two investment strategies


on part of the investor:

Buying all of firm B’s equity: The investor’s wealth is just enough to buy the
equity at the given market value. Since she would then be the sole equity
owner of the firm, she would receive the entire equity claims at t=1 which
from (a) we know to be 66.

Buy all of firm A’s equity: The investor needs to borrow 𝐸 − 𝑊 = 46 to buy
the entire equity stake at t=0. Since the firm is not leveraged, the investor
would receive 110 at t=1. Out of this, she would have to repay her personal
debt at the same rate as the firm: 46 ∙ (1 + 0.1) = 50.6. Thus, her final payoff
would be 110 − 50.6 = 59.4.

c) Clearly, firm B is undervalued. The most evident arbitrage strategy is therefore to


sell stock A short and use the proceeds to buy stock B.

Example: Short selling A equity at t=0 at the market value yields proceeds
100. Use 54 (= 𝐸 ) thereof to buy firm B’s equity and invest the rest,
(100 – 𝐸 ), in debt. At t=1, the investor gets cash flow from B, i.e. 66, and from
the safe debt, i.e. 46 ∙ (1 + 0.1) = 50.6, and hast to repay the cash flow on the
short-sold stocks of firm A, i.e. 110. Thus her payoff is 66 + 50.6 – 110 = 6.6.

Note that the investor needs no wealth to carry out this arbitrage investment.

If rational investors exploit all arbitrage opportunities, then the price of firm B
stock should rise until arbitrage is no longer possible. The value of firm B’s equity
at t = 0 must equal the discounted cash flows associated with it at t = 1, which
from (a) we know to be 60. The total firm values would then be
𝑉 = 𝑉 = 100
𝑉 = 𝑉 + 𝑉 = 60 + 40 = 100
and thus be identical for both firms.

This is an application of Modigliani Miller Proposition 1, which under the


appropriate assumptions states that the total value of a firm is independent of its
capital structure. See the lecture slides for a list of the assumptions.

Question 3:
∙ % %
a) 𝑟 = 𝑟 = + = 12%

b) i) 𝑟 = 𝑟 + (𝑟 − 𝑟 ) = 12% + (12% − 6%) = 18%

ii) If 𝑟 is higher, 𝑟 is lower. The debt will share some of the risk.

Question 4:
a) 𝑁𝑒𝑡 𝐼𝑛𝑐𝑜𝑚𝑒 = 𝐸𝐵𝐼𝑇 − 𝐼𝑛𝑡𝑒𝑟𝑒𝑠𝑡 − 𝑇𝑎𝑥𝑒𝑠 = (325 − 125)´(1 − 0.40) =
$120 𝑚𝑖𝑙𝑙𝑖𝑜𝑛.
b) $245 𝑚𝑖𝑙𝑙𝑖𝑜𝑛
c) 𝑁𝑒𝑡 𝑖𝑛𝑐𝑜𝑚𝑒 = 𝐸𝐵𝐼𝑇 − 𝑇𝑎𝑥𝑒𝑠 = 325´(1 − 0.40) = $195 𝑚𝑖𝑙𝑙𝑖𝑜𝑛. This is 245 −
195 = $50 𝑚𝑖𝑙𝑙𝑖𝑜𝑛 lower than in part (b).
d) 0.40 ∗ 125 = $50 𝑚𝑖𝑙𝑙𝑖𝑜𝑛.

Question 5:
a) FCF  EBIT  1     Dep  Capex  NWC  15  1  0.35   3  6  6.75
6.75
E  $450 million
10%  8.5%
15
b) Interest expense of $15 million  debt of  $187.5 million.
0.08

c) No. The most they should borrow is 187.5 million; there is no interest tax shield
from borrowing more.

Question 6:
150  135  95  80
a) 0.25   $109.52 million
1.05
100  100  95  80
b) 0.25   $89.28 million
1.05
100
c) YTM = – 1= 12%
89.29

expected return = 5%
50  35  0  0
d) equity = 0.25   $20.24 million
1.05

total value = 89.28 +20.24 = $109.52 million

Question 7:

a)
𝑟 = 5% + 1.1 ∙ (15% – 5%) = 16%

𝑉 = . = $100 million
b) Note how this exercises introduces cost of financial distress. COFD decreases
expected cash flows from 16 to 15 million, however, the cash flow risk is not
altered in an asymmetric or non-linear way. This assumption is somehow
restrictive, but from a modeling perspective the easiest to implement.

15
𝑉= + 0.35 ∙ 40 = $107.75 million
0.16

Question 8:

a) Exp(Value in one year) = 0.8 (50) + 0.2 (20) = 44.


𝐸 = 44/1.1 = $40𝑚
b) 𝐷 = 20/1.05 = 19.048.
Therefore, 𝐸 = 40 − 19.048 = $20.952𝑚
c) Without leverage, 𝑟 = 44/40 − 1 = 10%, with leverage, 𝑟 = (44 − 20)/
20.952 = 14.55%.
d) Without leverage, 𝑟 = 20/40 − 1 = −50%, with leverage, 𝑟 = . − 1 =
−100%
Problem Set 2
Corporate Finance 2.5 – Spring 2020
Question 1:

Zymase is a biotechnology start-up firm. Researchers at Zymase must choose one of


three different research strategies. The payoffs (after-tax) and their likelihood for each
strategy are shown below. The risk of each project is diversifiable.

STRATEGY PROBABILITY PAYOFF


A 100% 75
B 50% 140
50% 0
C 10% 300
90% 40

a) Which project has the highest expected payoff?


b) Suppose Zymase has debt of $40 million due at the time of the project's payoff.
Which project has the highest expected payoff for equity holders? Does the
ranking change if Zymase has debt of $110 million due at the time of the project's
payoff? How would you quantify agency cost?

Question 2:

Consider a firm that generates random cash flows at date 𝑡 = 1. Cash flows are either
200 or 100 with equal probability. At 𝑡 = 0 the firm has debt outstanding with a face
value of 150, which is due at 𝑡 = 1. The manager makes all investment decisions in the
interest of equityholders. All investors are risk-neutral, the discount rate is zero, and
there are no taxes.
a) What is the value of the firm at 𝑡 = 0? What is, respectively, the value of the firm’s
equity and the value of the firm’s debt at 𝑡 = 0?
b) Now assume that the firm has two mutually exclusive projects (Projects A and B).
Each project requires additional financing of 30 at 𝑡 = 0. If the firm invests in
Project A, the cash flows at 𝑡 = 1 will increase by 40 for sure. In other words, cash
flows will be either 240 or 140 with equal probability. If the firm invests in
Project B, cash flows increase by 50 in the high state and decrease by 50 in the
low state. That is, cash flows will either be 250 or 50 with equal probability.
Assume that there are covenants in place at 𝑡 = 0 that prohibit the issue of
additional debt. Which project would the manager choose if she had the money?
Are shareholders willing to provide the $30 for the investment?
c) The manager asks the debtholder to waive the covenants, so that the company
can issue senior debt. Assume that the manager only has the safe project (Project
A) available, and that the debtholders know this. Can the firm issue new debt of
$30 that is senior to the existing debt, to finance the project? Do the existing
debtholders agree to waive the seniority covenant?
d) Assume again that the manager asks the debtholder to waive the covenant.
Furthermore, debtholders now believe that there is a 50% chance that the
manager has both projects available. Would the company be able to issue new
senior debt to finance the project if existing debtholders waived the covenant?
Are existing debtholders willing to waive the seniority covenant?

Question 3:

Consider a risk neutral entrepreneur who plans a project that requires an investment of
$8 million at date 𝑡 = 0. At date 𝑡 = 1, the project generates cash flows of either $23
million or $5 million. The probability of success depends on the entrepreneur’s effort
choice. If the entrepreneur chooses “high effort”, the project succeeds with probability
80%. If the entrepreneur shirks, the project succeeds only with a 20% probability.
Exerting “high effort” is costly for the entrepreneur. Denote by 𝐶 the monetary
equivalent of this cost. Investors cannot observe the entrepreneur’s choice of effort.
a. Suppose the entrepreneur is able to finance the project with privately owned
funds. What is the NPV of the project if the entrepreneur chooses “high effort”?
What is the NPV instead if the entrepreneur chooses “low effort”?
b. Suppose the entrepreneur is penniless and intends to finance the investment by
issuing a fraction 𝛼 of equity to an outside investor. What fraction does the
entrepreneur have to sell to raise $8M? What is the maximum cost of effort 𝐶 that
is compatible with the entrepreneur choosing high effort?
c. Suppose the entrepreneur is penniless and intends to finance the investment
with a risky loan from a risk-neutral bank. What is the face value 𝐾 of the debt
claim if the entrepreneur raises $8M? What is the maximum cost of effort 𝐶 that
is compatible with the entrepreneur choosing high effort?
d. Discuss the relative merits of debt and equity finance by comparing the outcomes
of part b. and c.

Question 4:
Consider the following risky company. With probability ½ the true value of the firm’s
assets in place is $200. With probability ½ the true value of the firm’s assets in place is
only $50. Management is considering a new investment that would cost $20 and
generate a safe return of $25 next year (i.e. irrespective of the value of the assets in place
the investment will generate a cash flow of $25.) Suppose that the manager maximizes
the payoff for existing shareholders and that the discount rate is zero. The firm currently
has no debt outstanding,
a) Suppose that no one, not even the manager, knows the true value of the firm’s
assets in place. Everyone believes that the firm’s existing assets are worth $200
with probability ½ and $50 with probability ½. Will the manager undertake the
new investment? Will he prefer debt or equity financing? Explain.
b) Suppose that the true value of assets in place is $200, but only the manager
knows this. Everyone else still believes that the true value is $200 with
probability ½ and $50 with probability ½. Will the manager undertake the
project? If so, will he choose debt or equity financing? Explain.
c) How does your answer to b) change when the investment cost is $15 instead of
$20? Explain.

Question 5:
Consider a risk-neutral entrepreneur whose firm has assets in place that generate in
state 1 cash ow X1 = 80 and in state 2 cash ow X2 = 20 . Both states are equally likely,
that is, each state occurs with probability 0:5. The entrepreneur considers two
candidates for the CFO position, Mr. Bet and Mr. Hedge. Appointing Mr. Bet as CFO
increases the cash ow in state 1 by 10 and in state 2 by 0, whereas appointing Mr. Hedge
as CFO increases the cash ow in state 1 by 0 and in state 2 by 20.
a) Assume that the entrepreneur is the sole owner of the firm. Whom is she
choosing for the CFO position, Mr. Bet or Mr. Hedge?
b) Assume now - and for all remaining parts of the question - that the firm has
outstanding debt with face value K = 50. Does the entrepreneur select the same
candidate (as in a)) or not? Explain your findings.
c) What is the fair (market) value of the creditor's debt claim if Mr. Bet becomes
CFO? And what is the fair valuation if Mr. Hedge becomes CFO?
d) The entrepreneur offers creditors to convert their (entire) debt claim into an
equity claim. Assume all the bargaining power is with the entrepreneur, that is,
she can make a take-it-or-leave-it offer to the creditors. Which fraction of the
firm's equity is she offering in exchange for the debt? What interest or benefit
does the entrepreneur have in offering to convert debt into equity?
Solution – Problem Set 2

Corporate Finance 2.5 – Spring 2020


Question 1:

a) E(A) = $75 million


E(B) = 0.5 × 140 = $70 million
E(C) = 0.1 × 300 + 0.9 × 40 = $66 million
Project A has the highest expected payoff.
b) E(A) = 75 – 40 = $35 million
E(B) = 0.5 × (140 – 40) = $50 million
E(C) = 0.1 × (300 –40) + 0.9 × (40 – 40) = $26 million
Project B has the highest expected payoff for equity holders.
c) E(A) =$0 million
E(B) = 0.5 × (140 – 110) = $15 million
E(C) = 0.1 × (300 –110) = $19 million
Project C has the highest expected payoff for equity holders.
d) With $40 million in debt, management will choose project B, which has an expected
payoff for the firm that is 75 – 70 = $5 million less than project A. Thus, the expected
agency cost is $5 million.
With $110 million in debt, management will choose project C, resulting in expected
agency cost of 75– 66 = $9 million.

Question 2:

a) Firm value and payoff to debt and equity in the two states are

PROBABILITY FIRM VALUE DEBT EQUITY


HIGH STATE ½ 200 150 50
LOW STATE ½ 100 100 0

Hence, Firm value is


1 1
∙ 100 + ∙ 200 = 150
2 2
The value of debt is
1 1
∙ 100 + ∙ 150 = 125
2 2

And the value of equity is


1 1
∙ 0 + ∙ 50 = 25
2 2

b) Note: in the question it is not perfectly clear whether the firm has the cash or needs
outside financing. In the following calculations I assume the more interesting case
where the firm needs to raise outside financing. The conclusions with internal
financing are not the same. Verify it!

Back to the case of external financing: If project A is undertaken, the payoffs are

PROBABILITY FIRM VALUE DEBT EQUITY


HIGH STATE ½ 240 150 90
LOW STATE ½ 140 140 0
VALUE - 190 145 45

and if project B is undertaken

PROBABILITY FIRM VALUE DEBT EQUITY


HIGH STATE ½ 250 150 100
LOW STATE ½ 50 50 0
VALUE - 150 100 50

As management acts in the interest of equity, it will invest in project B if it has/had


the necessary funds of 30, even though project B is inferior and has a negative NPV.
This is an incidence of the asset substitution problem: By undertaking a risky
project, equityholders gain at the expense of the debtholders.
However, the equityholders would not be willing to bear the investment cost, i.e.,
put in an extra 30, as project B increases their return by only 25, relative to no
investment.

c) If the covenant is waived and the safe project A is undertaken, the payoffs are

PROBABILITY FIRM NEW OLD EQUITY


VALUE DEBT DEBT
HIGH STATE ½ 240 30 150 60
LOW STATE ½ 140 30 110 0
VALUE - 190 30 130 30

The new debt is senior and hence risk-free. Old debtholders benefit from the project
in the low state, and the value of debt increases from 125 to 130. Hence, the
debtholders would agree to waive the covenant.
d) The payoff if the covenant is waived, senior debt is issued, and project B is
undertaken are

PROBABILITY FIRM NEW OLD EQUITY


VALUE DEBT DEBT
HIGH STATE ½ 250 30 150 70
LOW STATE ½ 50 30 20 0
VALUE - 150 30 85 35
When project B is undertaken, new debt is also repaid in full. So there would not be
any problem raising senior debt once the covenant is waived.
As the equityholders are better off with project B than with project A (35 as opposed
to 30), management would always choose project B when available. Relative to the
no investment scenario, the value of old debt decreases to 85 as debtholders bear
the negative NPV of project B (30) plus the shareholders’ gain (10).
Given project B is available with probability ½, the value of the old debt is
1 1
∙ 130 + ∙ 85 = 107.5
2 2
As this is less than the value of debt with no investment (125), the debtholders will
not waive the covenant.

Question 3:

a) Compute expected values under high and low effort choice separately. (Don’t
confuse it with an asymmetric information exercise. Here, managers and investors
have all the same information set.)

𝑁𝑃𝑉 = 0.8 ∙ 23 + 0.2 ∙ 5 − 8 = 11.4


𝑁𝑃𝑉 = 0.2 ∙ 23 + 0.8 ∙ 5 − 8 = 0.6

b) Denote by 𝛼 the fraction of the firm that the entrepreneur sells to outside investors.
Assuming the entrepreneur exerts high effort, the participation constraint for
outside investors is given as follows:

𝛼 ∗ (0.8 ∗ 23 + 0.2 ∗ 5) ≥ 8 → 𝛼 = 0.41

The previous transaction is only feasible if it is credible that the entrepreneur


actually exerts effort. We have to verify this by looking at the entrepreneur’s IC
constraint:

(1 − 𝛼) ∗ (0.8 ∗ 23 + 0.2 ∗ 5) − 𝐶 ≥ (1 − 𝛼) ∗ (0.2 ∗ 23 + 0.8 ∗ 5) → 𝐶 ≤ 6.35

An entrepreneur can obtain equity financing if her effort cost 𝐶 is smaller than 6.35.
c) Denote by 𝐾 the face value of a risky bank loan that the entrepreneur can commit to
repay for borrowing 8 today. Again, assuming the entrepreneur exerts high effort,
the bank’s participation constraint is:

0.8 ∙ 𝐾 + 0.2 ∙ 5 ≥ 8 → 𝐾 = 8.75

We have to verify that the entrepreneur is willing to exert effort:

0.8 ∙ (23 − 8.75) − 𝐶 ≥ 0.2 ∙ (23 − 8.75) → 𝐶 ≤ 8.55


d) This exercise highlights the disciplining role of debt. Entrepreneur’s with 6.35 <
𝐶 < 8.55 cannot obtain equity financing. However, they can be financed through a
debt contract and investors’ are still breaking even.

Question 4:

a)
(i) Debt financing: The manager issues debt with face value $20. The expected
payoff of existing shareholders is:
0.5 ∙ (200 + 25) + 0.5 ∙ (50 + 25) − 20 = 150 − 20 = 130
(ii) Equity financing: Suppose the manager sells a stake x in the company to new
investors. The break-even constraint of new investor requires that
𝑥 ∙ 0.5 ∙ (200 + 25) + 𝑥 ∙ 0.5 ∙ (50 + 25) ≥ 20 ⇔ 𝑥 ≥ (2/15)
The manager would issue a stake x*=20/150
(iii) If no investment is made, shareholders expect to receive $125.
The manager will undertake the investment. Under both financing modes (i) and (ii)
existing shareholders receive an expected payoff of 130. Hence, the manager is
indifferent between the two choices.
b) The manager could still issue debt with face value $20 which allows new investors
to break even. Alternatively, the manager could choose equity financing in which
case he needs to issue an equity stake 𝑥 ∗ = 20/150 to new investors (as derived in
a)). By assumption the manager maximizes the payoff of existing shareholders.
Under debt financing, the payoff for existing shareholders is:
$200 + $25 − $20 = 205
Under equity financing, the payoff for existing shareholders is:
(1 − 2/15)(200 + 25) = 195
Hence the manager strictly prefers debt financing.
c) Under debt financing, existing shareholders receive $225– 15 = 210.
Under equity financing, new investors have the following break-even constraint:
15
𝑥 ∙ 0.5 ∙ (200 + 25) + 𝑥 ∙ 0.5 ∙ (50 + 25) ≥ 15 ⇔ 𝑥 ≥ = 0.1
150
Hence, the manager would sell a stake x*=10% to new investors. Existing shareholders
would receive (1 − 0.1) (200 + 25) = 202.5. The manager still prefers debt financing.

Question 5:

a) The expected cash flow when appointing Mr Bet is:


0.5 ∗ (80 + 10) + 0.5 ∗ (20) = 55
while appointing Mr Hedge results in expected cash flow
0.5 ∗ (80) + 0.5 ∗ (20 + 20) = 60
Hence, the entrepreneur chooses Mr. Hedge.
b) Debt remains risky, irrespective of whether Mr Bet or Mr Hedge is chosen as CFO.
Therefore, the entrepreneur does not take value enhancements in the low state 2
into account when taking the appointment decision. Appointing Mr Bet yields an
expected payoff to the entrepreneur of
0.5 ∗ (80 + 10 − 50)
while appointing Mr Hedge generates an expected payoff to the entrepreneur of
0.5 ∗ (80 − 50)
Hence, once risky debt is in place, the entrepreneur chooses to appoint Mr Bet.
c) When Mr Bet is the CFO, the creditors' expected payoff, that is, the market value of
debt is
𝐷 = 0.5 ∗ 50 + 0.5 ∗ 20 = 35
while under Mr Hedge as CFO the market value of debt is
𝐷 = 0.5 ∗ 50 + 0.5 ∗ 40 = 45
Thus, debtholders clearly prefer to see Mr. Hedge in office.
d) The debtholders know that if they keep their debt claim the entrepreneur is going to
appoint Mr. Bet. By contrast, when the firm is unlevered, the entrepreneur will
choose Mr Hedge. Given that the entrepreneur has all the bargaining power, it
suffices to offer creditors an equity stake such that they are indifferent between
keeping the debt claim or converting it into equity.
𝛼 ∗ (0.5 ∗ 80 + 0.5 ∗ 40) ≥ 35
𝛼 = 7/12
The entrepreneur in turn has an expected payoff of
7
1− ∗ 60 = 25
12
whereas without conversion his payoff is merely
0.5 ∗ (80 + 10 − 50) = 20
That is, the entrepreneur can appropriate the entire value increase from making the
efficient appointment decision.
Problem Set 3
Corporate Finance 2.5 – Spring 2020

Question 1:

Consider a firm that will operate for three years (Years 1, 2 and 3). The firm will generate
the following all-equity free cash flows (FCFs):

Year 1 Year 2 Year 3


Free Cash Flows to $100 $120 $110
all-equity firm

The company has risk free debt outstanding to the amount of $90 on which it makes annual
interest payments of 5%. The debt is only repaid (in full) at the end of Year 3. Suppose that
the firm’s asset beta is 1.5, the market risk premium is 6% and the risk free rate is 5%. The
corporate tax rate is assumed to be 40%. You want to value the firm in Year 0 using the
Adjusted Present Value (APV) Method.

a) What is the firm’s unlevered cost of capital?


b) What is the present value of the unlevered firm? Explain.
c) What is the present value of the tax shield? Explain.
d) What is the total value of the firm? Suppose you are offered to buy the company for
$255 in Year 0. Would you make the purchase? Explain.

Question 2:

Your firm is considering a $150 million investment to launch a new product line. The
project is expected to generate a free cash flow of $20 million per year, and its unlevered
cost of capital is 10%. To fund the investment, your firm will take on $100 million in
permanent debt.

a) Suppose the marginal corporate tax rate is 35%. Ignoring issuance costs, what is the
NPV of the investment?
b) Suppose your firm will pay a 2% underwriting fee when issuing the debt. It will
raise the remaining $50 million by issuing equity. In addition to the 5%
underwriting fee for the equity issue, you believe that your firm’s current share
price of $40 is $5 per share less than its true value. What is the NPV of the
investment including any tax benefits of leverage? (Assume all fees are on an after-
tax basis.)
Question 3:

You are a consultant who was hired to evaluate a new product line for Markum Enterprises.
The upfront investment required to launch the product line is $10 million. The product will
generate free cash flow of $750,000 the first year, and this free cash flow is expected to
grow at a rate of 4% per year. Markum has an equity cost of capital of 11.3%, a debt cost of
capital of 5%, and a tax rate of 35%. Markum maintains a debt-equity ratio of 0.40.

a) What is the NPV of the new product line (including any tax shields from leverage)?
b) How much debt will Markum initially take on as a result of launching this product
line?
c) How much of the product line’s value is attributable to the present value of interest
tax shields?

Question 4:

Acort Industries has 10 million shares outstanding and a current share price of $40 per
share. It also has long-term debt outstanding. This debt is risk-free, is four years away from
maturity, has annual coupons with a coupon rate of 10%, and has a $100 million face value.
The first of the remaining coupon payments will be due in exactly one year. The riskless
interest rates for all maturities are constant at 6%. Acort has EBIT of $106 million, which is
expected to remain constant each year. New capital expenditures are expected to equal
depreciation and equal $13 million per year, while no changes to net working capital are
expected in the future. The corporate tax rate is 40%, and Acort is expected to keep its
debt-equity ratio constant in the future (by issuing additional debt or buying back some
debt as times goes on).
a) Based on the information above, estimate Acort's WACC.
b) What is Acort's equity cost of capital?

Question 5:

For investment reasons you become interested in the valuation of firm A&Z. Since A&Z is
not publicly traded, you cannot rely on market prices. In your opinion, M&H is the most
similar publicly traded competitor that you intend to use as your benchmark. M&H
maintains a constant debt/value ratio of 20%. Financial media reports an equity beta of 0.8
and a risk-free debt beta. The corporate tax rate of both companies is 40%. The CAPM
holds with a risk-free rate of 3% and a market risk premium of 5%.
a) What is M&H’s asset beta? And what are its corresponding unlevered cost of capital?
A&Z maintains instead a debt/value ratio of 0.5, which implies a debt beta of 0.2. Analysts
forecast EBIT of $40 million for the next year growing at a rate of 2% p.a. in perpetuity.
Depreciation expenses balance capital expenditures and there are no changes to net
working capital.
b) What are A&Z’s weighted average cost of capital (WACC)?
c) What is the free cash flow of A&Z next year?
d) Based on the WACC method, compute the value of A&Z with leverage.
e) How much debt does A&Z have to issue/retire over the next year such that its debt-
equity ratio remains constant at the end of the year? What is the growth rate of
debt?
Solution - Problem Set 3
Corporate Finance 2.5 – Spring 2020
Question 1:

a) What is the unlevered cost of capital? 𝑟 = 5% + 1.5 ∙ 6% = 14%


b) What is the present value of the unlevered firm? Explain your calculations.
𝑃𝑉(𝑎𝑙𝑙 − 𝑒𝑞𝑢𝑖𝑡𝑦) = ( )
+( )
+ = 254
. . .
c) What is the present value of the tax shield? Explain your calculations.
. ∙ ∙ . . ∙ ∙ . . ∙ ∙ .
𝑃𝑉𝑇𝑆 = ( . )
+ ( . )
+ .
= 4.88
d) 𝐴𝑃𝑉 = 𝑉 = 𝑉 + 𝑃𝑉𝑇𝑆 = 254 + 4.88 = 258.8

Question 2:

a) With permanent debt the APV method is simplest:


𝑁𝑃𝑉(𝑢𝑛𝑙𝑒𝑣𝑒𝑟𝑒𝑑) =– 150 + 20/0.10 = $50 𝑚𝑖𝑙𝑙𝑖𝑜𝑛.
𝑃𝑉𝑇𝑆 = 𝜏 ∙ 𝐷 = 35% ∙ 100 = $35 𝑚𝑖𝑙𝑙𝑖𝑜𝑛.
The NPV with leverage is 𝐴𝑃𝑉 = 𝑁𝑃𝑉 + 𝑃𝑉𝑇𝑆 = 50 + 35 = $85 𝑚𝑖𝑙𝑙𝑖𝑜𝑛.
b) 𝐹𝑖𝑛𝑎𝑛𝑐𝑖𝑛𝑔 𝑐𝑜𝑠𝑡𝑠 (𝑎𝑓𝑡𝑒𝑟 𝑡𝑎𝑥) = (2% ∙ 100 + 5% ∙ 50)(1 − 35%) = $2.925 𝑚𝑖𝑙𝑙𝑖𝑜𝑛.
𝑈𝑛𝑑𝑒𝑟𝑝𝑟𝑖𝑐𝑖𝑛𝑔 𝑐𝑜𝑠𝑡 = (50/40) ∙ 5 = $6.25 𝑚𝑖𝑙𝑙𝑖𝑜𝑛
𝐴𝑃𝑉 = 85– 2.925– 6.25 = 75.825 𝑚𝑖𝑙𝑙𝑖𝑜𝑛

Question 3:
.
a) 𝑊𝐴𝐶𝐶 = ∙ 11.3% + ∙ 5% ∙ (1– 0.35) = 9%
. .
.
𝑉 = = $15 𝑚𝑖𝑙𝑙𝑖𝑜𝑛
% %
𝑁𝑃𝑉 = −10 + 15 = $5 𝑚𝑖𝑙𝑙𝑖𝑜𝑛
.
b) The Debt-to-Value ratio is .
= 28.57%.
Therefore Debt is 28.57% ∙ $15 𝑚𝑖𝑙𝑙𝑖𝑜𝑛 = $4.29 𝑚𝑖𝑙𝑙𝑖𝑜𝑛.
.
c) Discounting at 𝑟 gives unlevered value. 𝑟 = ∙ 11.3% + ∙ 5% = 9.5%
. .
.
𝑉 = = $13.64 𝑚𝑖𝑙𝑙𝑖𝑜𝑛
. % %

Tax shield value is therefore 15– 13.64 = 1.36 𝑚𝑖𝑙𝑙𝑖𝑜𝑛.


Alternatively, initial debt is $4.29 million, for a tax shield in the first year of 4.29 ∙
5% ∙ 0.35 = 0.075 𝑚𝑖𝑙𝑙𝑖𝑜𝑛.
.
Then 𝑃𝑉𝑇𝑆 = . %– %
= 1.36 𝑚𝑖𝑙𝑙𝑖𝑜𝑛.
Question 4:

a) We don’t know Acort’s equity cost of capital, so we cannot calculate WACC directly.
However, we can compute it indirectly by estimating the discount rate that is
consistent with Acort’s market value. First, 𝐸 = 10 ∙ 40 = $400 𝑚𝑖𝑙𝑙𝑖𝑜𝑛. The market
value of Acort’s debt is D = 10 ∙ 1− + = $113.86 𝑚𝑖𝑙𝑙𝑖𝑜𝑛.
. . .
Therefore, Acort’s enterprise value is 𝐸 + 𝐷 = 400 + 113.86 = 513.86.
Acort’s 𝐹𝐶𝐹 = 𝐸𝐵𝐼𝑇 ∙ (1 − 𝜏 ) + 𝐷𝑒𝑝 − 𝐶𝑎𝑝𝑒𝑥 − 𝐶ℎ𝑎𝑛𝑔𝑒 𝑖𝑛 𝑁𝑊𝐶.
𝐹𝐶𝐹 = 106 ∙ (1 − 0.4) = 63.6. Because Ascort is not expected to grow, 𝑉 =
.
513.86 = and so 𝑟 = 12.38%
.
b) Using 𝑟 = 𝑟 + 𝑟 (1 − 𝜏 ), 12.38% = .
𝑟 + .
6%(1 − 0.4)
Solving for 𝑟 = 14.88%.

Question 5:

a) 𝛽 = 0.2 ∙ 0 + 0.8 ∙ 0.8 = 0.64 𝑟 = 3% + 0.64 ∙ 5% = 6.2%


b) 𝛽 = 0.64 + 1 ∗ (0.64 − 0.2) = 1.08 𝑟 = 3% + 1.08 ∙ 5% = 8.4%
𝑟 = 3% + 0.2 ∙ 5% = 4% 𝑊𝐴𝐶𝐶 = 0.5 ∗ 8.4% + 0.5 ∗ (1 − 0.4) ∙ 4% = 5.4%
c) 𝐹𝐶𝐹 = $40𝑀 ∙ (1 − 0.4) = $24𝑀
$
d) 𝑉 = . .
= $705.88𝑀
$ ∗ .
e) 𝑉 = = $720𝑀 𝐷 = 0.5 ∗ $705.88𝑀 = $352.94𝑀
. .
𝐷 = 0.5 ∗ $720𝑀 = $360𝑀 𝐷𝑒𝑙𝑡𝑎 𝐷 = $7.06𝑀 → 2%
Problem Set 4
Corporate Finance 2.5 – Spring 2020

Question 1:

KMS Corporation has assets with a market value of $500 million, $50 million of which are
cash. It has debt of $200 million, and 10 million shares outstanding. Assume perfect capital
markets.

a) What is its current stock price?


b) If KMS distributes $50 million as a dividend, what will its share price be after the
dividend is paid?
c) If instead, KMS distributes $50 million as a share repurchase, what will its share
price be once the shares are repurchased?
d) What will its new market debt-equity ratio be after either transaction?
Question 2:
AMC Corporation currently has an enterprise value of $400 million and $100 million in
excess cash. The firm has 10 million shares outstanding and no debt. Suppose AMC uses its
excess cash to repurchase shares. After the share repurchase, news will come out that will
change AMC’s enterprise value to either $600 million or $200 million.
a) What is AMC’s share price prior to the share repurchase?
b) What is AMC’s share price after the repurchase if its enterprise value goes up? What
is AMC’s share price after the repurchase if its enterprise value declines?
c) Suppose AMC waits until after the news comes out to do the share repurchase. What
is AMC’s share price after the repurchase if its enterprise value goes up? What is
AMC’s share price after the repurchase if its enterprise value declines?
d) Suppose AMC’s management expects good news to come out, will they undertake
the repurchase before or after the news comes out? When would management
undertake the repurchase if they expect bad news to come out? What effect would
an announcement of a share repurchase have on the stock price?

Question 3:

Suppose Lucent Technologies has an equity cost of capital of 10%, market capitalization of
$10.8 billion, and an enterprise value of $14.4 billion. Suppose Lucent’s debt cost of capital
is 6.1% and its marginal tax rate is 35%.

a) What is Lucent’s WACC?


b) If Lucent maintains a constant debt-equity ratio, what is the value of a project with
average risk and the following expected free cash flows?
Year 0 1 2 3
FCF -100 50 100 70
c) If Lucent maintains its debt-equity ratio, what is the debt capacity of the project in
part (b)?
d) What is the free cash flow to equity for this project?
e) What is its NPV computed using the FTE method? How does it compare with the
NPV based on the WACC method?
Question 4:

Lockhead, one of the largest defense contractors in the United States, reported EBITDA of
$1,290 million in a recent financial year, prior to interest expenses of $215 million and
depreciation charges of $400 million. Capital expenditures amounted to $450 million
during the year, and working capital was 7% of revenues (which were $13,500 million).
The firm had debt outstanding of $3.068 billion (in book value terms), trading at a market
value of $3.2 billion, and yielding a pretax interest rate of 8%. There were 62 million shares
outstanding, trading at $64 per share, and the most recent beta is 1.10. The tax rate for the
firm is 40%. (The treasury bond rate is 7% and the market risk premium 5.5%.) The firms
expects revenues, earnings, capital expenditures, depreciation, and net working capital to
grow at 9.5% a year for the next five years, after which the growth rate is expected to drop
to 4%. (You can assume that capital spending will offset depreciation in the stable-growth
period.) The company also plans to lower its debt/equity ratio to 50% for the steady state
(which will result in the pretax interest rate dropping to 7.5%).

a. Estimate the value of the firm.


b. Estimate the value of the equity in the firm and the value per share.

Question 5:

Prokter and Gramble (PG) has historically maintained a debt-equity ratio of approximately
0.20. Its current stock price is $50 per share, with 2.5 billion shares outstanding. The firm
enjoys very stable demand for its products, and consequently it has a low equity beta of
0.50 and can borrow at 4.20%, just 20 basis points over the risk-free rate of 4%. The
expected return of the market is 10%, and PG’s tax rate is 35%.

a) This year, PG is expected to have free cash flows of $6.0 billion. What constant
expected growth rate of free cash flow is consistent with its current stock price?
b) PG believes it can increase debt without any serious risk of distress or other costs.
With a higher debt-equity ratio of 0.50, it believes its borrowing costs will rise only
slightly to 4.50%. If PG announces that it will raise its debt-equity ratio to 0.5
through a leveraged recap, determine the increase in the stock price that would
result from the anticipated tax savings.
Question 6:

In year 1, AMC will earn $2000 before interest and taxes. The market expects these
earnings to grow a rate of 3% per year. The firm will make no net investments (i.e., capital
expenditures will equal depreciation) or changes to net working capital. Assume that the
corporate tax rate equals 40%. Right now, the firm has $5000 in risk-free debt. It plans to
keep a constant ratio of debt to equity every year, so that on average the debt will also
grow by 3% per year. Suppose the risk-free rate equals 5%, and the expected return on the
market equals 11%. The asset beta for this industry is 1.11.

a) If AMC were an all-equity (unlevered) firm, what would its market value be?
b) Assuming the debt is fairly priced, what is the amount of interest AMC will pay next
year? If AMC’s debt is expected to grow by 3% per year, at what rate are its interest
payments expected to grow?
c) Even though AMC’s debt is riskless (the firm will not default), the future growth of
AMC’s debt is uncertain, so the exact amount of the future interest payments is
risky. Assuming the future interest payments have the same beta as AMC’s assets,
what is the present value of AMC’s interest tax shield?
d) Using the APV method, what is AMC’s total market value, 𝑉 ? What is the market
value of AMC’s equity?
e) What is AMC’s WACC? (Hint: Work backward from the FCF and 𝑉 .)
f) Using the WACC method, what is the expected return for AMC equity?
g) Show that the following holds for AMC: 𝛽 = 𝛽 +𝛽
h) Assuming that the proceeds from any increases in debt are paid out to equity
holders, what cash flows do the equity holders expect to receive in one year? At
what rate are those cash flows expected to grow? Use that information plus your
answer to part (f ) to derive the market value of equity using the FTE method. How
does that compare to your answer in part (d)? dividend?
Solution - Problem Set 4
Corporate Finance 2.5 – Spring 2020

Question 1:
a) Current stock price: (500 – 200)/10 = 30
b) Stock price after dividend: (450 – 200)/10 = 25
c) Stock price after repurchase: (450 – 200)/(10 – 1.667) = 30
d) Debt-equity ratio: 200/250 = 0.8

Question 2:
a) Equity = EV + Cash - Debt = $500 million.
Share price = ($500 million) / (10 million shares) = $50 per share.
b) AMC repurchases $100 million / ($50 per share) = 2 million shares. With 8 million
remaining share outstanding (and no excess cash) its share price if its EV goes up to
$600 million is Share price = $600 / 8 = $75 per share.
And if EV goes down to $200 million: Share price = $200 / 8 = $25 per share.
c) If EV rises to $600 million prior to repurchase, given its $100 million in cash and 10
million shares outstanding, AMC’s share price will rise to:
Share price = (600 + 100) / 10 = $70 per share.
If EV falls to $200 million:
Share price = (200 + 100) / 10 = $30 per share
The share price after the repurchase will be also be $70 or $30, since the share
repurchase itself does not change the stock price.
d) If management expects good news to come out, they would prefer to do the repurchase
first, so that the stock price would rise to $75 rather than $70. On the other hand, if they
expect bad news to come out, they would prefer to do the repurchase after the news
comes out, for a stock price of $30 rather than $25. (Intuitively, management prefers to
do a repurchase if the stock is undervalued—they expect good news to come out —but
not when it is overvalued because they expect bad news to come out.)
We expect managers to do a share repurchase before good news comes out and after
any bad news has already come out. Therefore, if investors believe managers are better
informed about the firm’s future prospects, and that they are timing their share
repurchases accordingly, a share repurchase announcement would lead to an increase
in the stock price.

Question 3:
. . .
a) 𝑟 = .
∙ 10% + .
∙ (1 − 0.35) ∙ 6.1% = 8.49%
b) Using the WACC method, the levered value of the project at date 0 is
50 100 70
𝑉 = + + = 185.86
1.0849 1.0849 1.0849
Given a cost of 100 to initiate, the project’s NPV is 185.86 – 100 = 85.86.
c) Lucent’s debt-to-value ratio is 𝑑 = (14.4 – 10.8) / 14.4 = 0.25. The project’s debt
capacity is equal to d times the levered value of its remaining cash flows at each date.
Year 0 1 2 3
FCF -100 50 100 70
𝑽𝑳 185.86 151.64 64.52 0
𝑫 = 𝒅 ∙ 𝑽𝑳 46.47 37.91 16.13 0

d) Using the debt capacity calculated in c), we can compute FCFE by adjusting FCF for
after-tax interest expense ((1 − 𝜏 ) ∙ 𝑟 ∙ 𝐷) and net borrowing (𝐷 − 𝐷 ).

Year 0 1 2 3
D 46.47 37.91 16.13 0
FCF -100 50 100 70
After-tax interest expense 0 -1.84 -1.5 -0.64
Net borrowing 46.47 -8.55 -21.78 -16.13
FCFE -53.53 39.6 76.72 53.23

. . .
e) 𝑃𝑉(𝐹𝐶𝐹𝐸) = −53.53 + + + = 85.86
. . .

Question 4:
0 1 2 3 4 5 Terminal
Revenues 13500 14782.50 16186.84 17724.59 19408.42 21252.22 22102.31
EBITDA 1290 1412.55 1546.74 1693.68 1854.58 2030.77 2112.00
Depreciation 400 438.00 479.61 525.17 575.06 629.70 654.88
CapEx 450 492.75 539.56 590.82 646.95 708.41 654.88
Working Capital 945 1034.78 1133.08 1240.72 1358.59 1487.66 1547.16
FCF 440.21 482.02 527.82 577.96 632.87 814.76
PV(FCF) 402.50 402.99 403.48 403.97 404.46
a) 𝐹𝐶𝐹 = (𝐸𝐵𝐼𝑇𝐷𝐴 − 𝐷𝑒𝑝𝑟)(1 − 𝑡) − (𝐶𝑎𝑝𝐸𝑥 − 𝐷𝑒𝑝𝑟) − ∆𝑁𝑊𝐶
Capital structure: 𝐸 = 64 ∗ 62 = 3968 and 𝑉 = 3200 + 3968 = 7168
Cost of equity: 𝑟 = 7% + 1.1 ∗ 5.5% = 13.05%
3968 3200
𝑊𝐴𝐶𝐶 = ∗ 13.05% + ∗ (1 − 0.4) ∗ 8% = 9.37%
7168 7168
Cost of capital changes for terminal value due to different capital structure.
Unlevered cost of capital: 𝑟 = ∗ 13.05% + ∗ 8% = 10.8%
New cost of equity: 𝑟 = 10.8% + 0.5 ∗ (10.8% − 7.5%) = 12.44%
New WACC: 𝑊𝐴𝐶𝐶 = ∗ 12.44% + ∗ (1 − 0.4) ∗ 7.5% = 9.8%
Terminal Value (at t=5)=814.76/(9.8%-4%)=14058.46
440.21 482.02 527.82 577.96 632.87 14058.46
𝑉 = + + + + + = 11002.17
1.0937 1.0937 1.0937 1.0937 1.0937 1.0937
Value of firm=11002, Equity=11002-3200=7802, price per share=125.84
According to our estimation, the shares are grossly underpriced. This is the moment when we
would start fine-tuning our model by testing the sensitivity of the estimation with respect to the
assumptions used. One problem may be that we allow our terminal value to grow forever at a
4% rate without increasing invested capital.

Question 5:
a) 𝐸 = $50 ∙ 2.5𝐵 = $125𝐵
𝐷 = 0.20 ∙ 125𝐵 = $25𝐵
𝑉 = 𝐸 + 𝐷 = $150𝐵
From CAPM: Equity Cost of Capital = 4% + 0.5(10%– 4%) = 7%
𝑊𝐴𝐶𝐶 = ∙ 7% + ∙ 4.2% ∙ (1 − 35%) = 6.29%
𝑉 = ⇒𝑔 =𝑟 − = 6.29% − = 2.29%

b) Initial Unlevered cost of capital (Eq. 18.6) = 7% + 4.2% = 6.53%


New Equity cost of capital (Eq. 18.10) = 6.53% + 0.5 ∙ (6.53% − 4.5%) = 7.55%
.
New WACC = ∙ 7.55% + ∙ 4.5% ∙ (1 – 35%) = 6.01%
. .
.
𝑉 = = = 161.29
. % . %
.
This is a gain of 161.29 − 150 = $11.29𝐵 or .
= $4.52 per share.
Thus, share price rises to $54.52/share.

Question 6:
a) AMC has unlevered FCF of $2,000 ∙ 0.6 = $1,200.
From the CAPM, AMC’s unlevered cost of capital is 5% + 1.11 ∙ (11% − 5%) = 11.66%
Discounting the FCF as a growing perpetuity tells us that the value of the firm, assuming
growth of 3%, is:

$1,200
𝑉(𝐴𝑙𝑙 𝐸𝑞𝑢𝑖𝑡𝑦) = = $13,857.
0.1166 − 0.03
b) Since the debt is risk-free, the interest rate paid on it must equal the risk-free rate of 5%
(or else there would be an arbitrage opportunity). The firm has $5,000 of debt next
year. The interest payment will be 5% of that, or $250. If the debt grows by 3% per
year, so will the interest payments.
c) The expected value of next year’s tax shield will be $250 × 40% = $100, and it will grow
(with the growth of the debt) at a rate of 3%. But the exact amount of the tax shield is
uncertain, since AMC may add new debt or repay some debt during the year, depending
on their cash flows. This makes the actual amount of the tax shield risky (even though
the debt itself is not). Since the beta of the tax shield due to debt is 1.11, the appropriate
discount rate is 5% + 1.11 (11% – 5%) = 11.67%. We can now use the growing
perpetuity formula and conclude that

$100
𝑃𝑉𝑇𝑆 = = $1,155.
0.1166 − 0.03
d) The APV tells us that the value of a firm with debt equals the sum of the value of an all
equity firm and the tax shield. From previous work (parts (a) and (c)), we get:
𝑉(𝐴𝑀𝐶) = $13,857 + $1,155 = $15,012.
The market value of the equity is therefore 𝑉 – 𝐷 = $15,012 − $5000 = $10,012.
e) Next year’s FCF is $2,000 ∙ 0.6 = $1,200. It is expected to grow at 3%, so the WACC must
satisfy:
$1,200
𝑉(𝐴𝑀𝐶) = = $15,012.
𝑟 − 0.03
Solving for the WACC, we get 𝑟 ≈ 11%.
f) By definition, 𝑟 = ∙ 𝑟 + ∙ 𝑟 ∙ (1 − 𝜏 ).
The return on the debt is 5%; the value of the debt is $5,000, the value of the firm is
$15,012 and therefore the value of the equity is $15,012– $5,000 = $10,012. Plugging
into the above expression, we get:
$10,012 $5,000
11% = ∙𝑟 + ∙ 5% ∙ (1 − 0.4) ⇒ 𝑟 ≈ 15%.
$15,012 $15,012
g) From the CAPM, 𝛽 must satisfy 15% = 5% + 𝛽 ∙ (11% − 5%), so we conclude 𝛽 =
1.66.
The relationship holds since ($10,012/$15,012) ∙ 1.66 ≈ 1.11, and the beta of the debt
equals 0.
h) The debt is expected to increase to $5,000 ∙ (1 + 0.03) = $5,150, so the equity holders
will get $150 due to the increase in debt. These proceeds will increase by 3% annually.
(The second-year debt will be $5,000 ∙ (1 + 0.03) = $5,304.5, with an increase in debt
of $154.5, 3% higher than the $150 proceeds of year 1.) The expected FCF to equity at
the end of the first year is therefore EBIT – Interest – Taxes + Debt proceeds, or 𝐹𝐶𝐹𝐸 =
(2000 − 250) ∙ (1 − 0.40) + 150 = $1200.
This cash flow is expected to grow at 3% per year. Thus, another way to compute the
value of equity is to discount these cash flows at the equity cost of 15% (from (f)):
𝐹𝐶𝐹𝐸 $1,200
𝐸= = = $10,000 ≈ $10,012
𝑟 − 𝑔 15% − 3%
Note: This is the same value we computed in (d), using the APV. The small difference is
due to the rounding of 𝑟 .
Problem Set 5
Corporate Finance 2.5 – Spring 2020
Question 1:

Three years ago, you founded your own company. You invested $100,000 of your
money and received 5 million shares of Series A preferred stock. Since then, your
company has been through three additional rounds of financing.

Round Price ($) Number of Shares


Series B 0.50 1,000,000
Series C 2.00 500,000
Series D 4.00 500,000
a) What is the pre-money valuation for the Series D funding round?
b) What is the post-money valuation for the Series D funding round?
c) Assuming that you only own the Series A preferred stock (and that each share of
all series of preferred stock is convertible into one share of common stock), what
percentage of the firm do you own after the last funding round?

Question 2:

Your firm has 10 million shares outstanding, and you are about to issue 5 million new
shares in an IPO. The IPO price has been set at $20 per share, and the underwriting
spread is 7%. The IPO is a big success with investors, and the share price rises to $50
the first day of trading.

a) How much did your firm raise from the IPO?


b) What is the market value of the firm after the IPO?
c) Assume that the post-IPO value of your firm is its fair market value. Suppose
your firm could have issued shares directly to investors at their fair market
value, in a perfect market with no underwriting spread and no underpricing.
What would the share price have been in this case, if you raise the same amount
as in part a)?
d) Comparing part b) and part c), what is the total cost to the firm’s original
investors due to market imperfections from the IPO?

Question 3:

Suppose P&G is considering purchasing $15 million in new manufacturing equipment. If


it purchases the equipment, it will depreciate it on a straight-line basis over the five
years, after which the equipment will be worthless. It will also be responsible for
maintenance expenses of $1 million per year. Alternatively, it can lease the equipment
for $4.2 million per year for the five years, in which case the lessor will provide
necessary maintenance. Assume P&G’s tax rate is 35% and its borrowing cost is 7%.
a) What is the NPV associated with leasing the equipment versus financing it with
the lease-equivalent loan?
b) What is the break-even lease rate – that is, what lease amount could P&G pay
each year and be indifferent between leasing and financing a purchase?

Question 4:

Consider a firm with a partially concentrated ownership structure. A risk neutral large
shareholder owns a fraction 𝛼, while the remaining 1 − 𝛼 shares are dispersed among
small risk-neutral investors. The firm is run by a risk neutral manager whose effort
choice 𝑒 ∈ {0, 1} affects firm performance. If the manager works (𝑒 = 1) at a cost c = 10
the firm generates cash flow 𝑋 = 100, whereas if she shirks (𝑒 = 0) cash flow is 𝑋 =
40. Assume that the manager is offered some fixed wage 𝑤. Finally, any shareholder can
engage in costly monitoring to ensure that the manager works. Monitoring entails a cost
𝑚 = 5 which is borne by the monitoring shareholder.
a) Given the manager anticipates that she will be monitored, what fixed salary 𝑤
induces the manager to accept the job. How large an equity stake 𝛼 does a
shareholder need to have in order to monitor the manager?
b) Propose an incentive (performance-based) contract that ensures that the
manager works in the absence of monitoring. Do shareholders prefer offering the
manager an incentive contract relative to the fixed wage and monitoring (result
found in a)? Explain.
c) Assume now that if the manager shirks (𝑒 = 0) cash flow is 𝑋 = 100 with
probability 1/2 and 𝑋 = 40 with probability 1/2. (If the manager works the
cash flow is 𝑋 = 100 with certainty.) How does this affect the incentive
contract?
d) Based on the above discussion, will shareholders offer a fixed wage and monitor
or an incentive contract? Phrased differently, is total shareholder wealth higher
under a) or c)?

Question 5:

Consider an all-equity firm with a partially concentrated ownership structure in a one-


period model. A large shareholder owns 10% of shares, while the remaining shares are
dispersed among small investors. The firm generates cash flows of 𝑋 = 140 or 𝑋 =
40. The firm is run by a manager whose effort choice affects the firm’s success
probability. If the manager puts effort into the project (at a cost 𝑐 = 5), the success
probability is 𝑝 = 0.6, otherwise 𝑝 = 0.2. Finally, the large shareholder can engage in
costly monitoring to ensure that the manager works with effort (at a cost 𝑚 = 5), which
cannot be split among shareholder. Everyone is risk-neutral and there is no discounting.
a) Assume that that the manager is offered some fixed wage 𝑤. Given that the
manager anticipates that she will be monitored, what fixed salary 𝑤 induces the
manager to accept the job. Will the large shareholder be willing to monitor?
What do you conclude?
b) Suppose now that shareholders decide to compensate the manager with stock in
the company instead of a wage. What fraction of the firm do shareholders have to
offer such that the manager prefers to exert effort? Would the large shareholder
prefer this over the fixed wage solution found in (a)? What is the expected value
of small shareholders? And what is the expected value of the manager’s stake?
c) Suppose instead that the shareholders decide to offer a performance-based
contract to the manager that ensures that the manager works with effort. How
would this contract look like? Does the large shareholders prefer this option over
the solution in (b)? How about small shareholders? And what is the expected
value of the manager’s contract?
d) Finally, shareholders decide to issue risk-free debt with a face value of 40. (The
proceeds are used to repurchase shares from existing shareholders.) What
fraction of the levered equity do shareholders have to offer to the manager in
this case such that it is incentive compatible (recalculate (b))? What will be the
new performance-based contract (from (c))? Compare the two expected values
of the manager’s claims. What can you conclude?
Solution - Problem Set 5
Corporate Finance 2.5 – Spring 2020

Question 1:
Before the Series D funding round, there are 6.5 million shares outstanding. Given a Series
D funding price of $4.00 per share, the pre-money valuation is

$4.00
6,500,000 𝑠ℎ𝑎𝑟𝑒𝑠 ∙ = $26 𝑚𝑖𝑙𝑙𝑖𝑜𝑛
𝑠ℎ𝑎𝑟𝑒
a) After the funding round there will be 7 million shares outstanding, so the post-
money valuation is $28 million.
b) You will own = 71.4% of the firm after the last funding round.

Question 2:
a) 5𝑀 ∙ $20 ∙ (1 − 0.07) = $93𝑀
b) 𝑉 = 15𝑀 ∙ $50 = $750𝑀
c) Market value without newly raised cash is: 𝑉 = $750𝑀 − $93𝑀 = $657𝑀
$
The share price is then 𝑃 = = $65.70
d) Underpricing: ($65.70 − $50) ∙ 10𝑀 = $157𝑀

Question 3:
a) If P&G buys the equipment, it will pay $15𝑀 upfront, and have depreciation
expenses of $3𝑀 per year, generating a depreciation tax shield of 35% ∙ 3 = $1.05𝑀
per year for year 1-5. It will also have after-tax maintenance expenses of 1 ∙
(1 − 0.35) = 0.65𝑀. Thus the incremental FCF from buying is 1.05 − 0.65 = $0.4𝑀
in years 1-5.
If it leases, the after-tax lease payments are $4.2𝑀 ∙ (1 − 0.35) = $2.73𝑀. Thus, the
incremental FCF of leasing versus buying is −$2.73𝑀 − (−$15𝑀) = $12.27𝑀 in
year 0, −$2.73𝑀 − $0.4𝑀 = −$3.13𝑀 in years 1-4, and 0 − $0.4𝑀 = −$0.4𝑀 in
year 5. We can determine the gain from leasing by discounting the incremental cash
flows at P&G’s after-tax cost of borrowing rate of 7% ∙ (1 − 0.35) = 4.55%:

3.13 3.13 3.13 3.13 0.4


𝑁𝑃𝑉(𝐿𝑒𝑎𝑠𝑒 − 𝑏𝑢𝑦) = 12.27 − − − − −
1.0455 1.0455 1.0455 1.0455 1.0455
= $733,955
Under these assumptions, the lease is more attractive than financing a purchase of
the computer.
b) We can increase the after-tax lease payments by an amount with present value
equal to the NPV in (a).

1 1
733,955 = ∆𝐿 ∙ (1 − 0.35) ∙ 1 + 1−
0.0455 1.0455
Therefore, the increase in lease payments $246,363. The break-even lease rate is
$4.2𝑀 + $0.246363𝑀 = $4.446363𝑀 per year.

Question 4:
a) The manager’s participation constraint is 𝑤 − 𝑐(𝑒) ≥ 0 ↔ 𝑤 − 10 ≥ 0. Hence, the
fixed wage has to be 𝑤 = 10 to induce the manager to accept the job, given she
anticipates to be monitored.
The large shareholder monitors if
𝛼(𝑉 − 𝑤) − 5 ≥ 𝛼𝑉 ↔ 𝛼(100 − 10) − 5 ≥ 𝛼40
𝛼(90 − 40) ≥ 5 ↔ 𝛼 ≥ 1/10
Note: I assume that shareholders will offer a wage of 𝑤 = 0 if the blockholder
decides not to monitor (which is the equilibrium where we do not expect managers
to exert effort).
b) Since firm performance is perfectly linked to managerial effort the optimal
performance-based contract is simply:
10 + 𝜀 𝑖𝑓 𝑉 = 100
𝑤 ∗ (𝑉) =
0 𝑖𝑓 𝑉 = 40
Since performance is perfectly linked to effort an incentive contract solves the
agency problem at no cost. That is, total shareholder return is V − c = 90. By
contrast, the fixed wage and monitoring solution require the same wage (otherwise
the manager does not take the job) and additional monitoring costs of 5. (Strictly
speaking, small shareholders are indifferent as the monitoring costs are borne by
the large shareholders.)

c) The manager’s incentive compatibility constraint is


𝑤(𝑉 = 100) − 10 ≥ 0.5 ∙ 𝑤(𝑉 = 100)
which yields
𝑤(𝑉 = 100) ≥ 20
Hence, the optimal contract is
20 𝑖𝑓 𝑉 = 100
𝑤 ∗ (𝑉) =
0 𝑖𝑓 𝑉 = 40
and the shareholders total payoff if this contract is implemented is given by:

(100 − 20) = 80
d) Under monitoring the security value of the firm is: 100 − 10 = 90

Note that now the small shareholders prefer the monitoring solution, while the large
shareholder prefers the incentive contract unless his stake is such that

𝛼 ∙ 80 < 𝛼 ∙ 90 − 5 ↔ 𝛼 > 1/2

Question 5:
a)
𝑤−𝑐 ≥0 → 𝑤 =5
Large shareholder value monitoring: 0.1 ∗ (0.6 ∗ 140 + 0.4 ∗ 40 − 5) − 5 = 4.5
Large shareholder value not monitoring: 0.1 ∗ (0.2 ∗ 140 + 0.8 ∗ 40) = 6

Monitoring under a fixed contract is not feasible with such a small stake.
b) Manager’s IC constraint:
𝛼(0.6 ∗ 140 + 0.4 ∗ 40) − 5 ≥ 𝛼(0.2 ∗ 140 + 0.8 ∗ 40) → 𝛼 = 12.5%
Large shareholder value: 0.1 ∗ (1 − 0.125) ∗ (0.6 ∗ 140 + 0.4 ∗ 40) = 8.75
Small shareholder value: 0.9 ∗ (1 − 0.125) ∗ (0.6 ∗ 140 + 0.4 ∗ 40) = 78.75
Manager’s value: 0.125 ∗ (0.6 ∗ 140 + 0.4 ∗ 40) = 12.5

c) Don’t offer the manager anything if the bad state realizes.


0.6𝑤 − 5 ≥ 0.2𝑤 → 𝑤 = 12.5 𝑖𝑓 𝑋 = 140
Large shareholder value: 0.1 ∗ (0.6 ∗ (140 − 12.5) + 0.4 ∗ 40) = 9.25
Small shareholder value: 0.9 ∗ (0.6 ∗ (140 − 12.5) + 0.4 ∗ 40) = 83.25
Manager’s expected value: (0.6 ∗ 12.5) = 7.5
d) Manager’s IC constraint:
𝛼 0.6 ∗ (140 − 40) + 0.4 ∗ (40 − 40) − 5
≥ 𝛼 0.2 ∗ (140 − 40) + 0.8 ∗ (40 − 40)
→ 𝛼 = 12.5%
The performance-contract does not change!
Manager’s value under equity: 0.125 ∗ (0.6 ∗ (140 − 40) + 0.4 ∗ (40 − 40)) = 7.5
Manager’s value under performance contract: (0.6 ∗ 12.5) = 7.5

You might also like